Đến nội dung

Prudential112410 nội dung

Có 423 mục bởi Prudential112410 (Tìm giới hạn từ 20-04-2020)



Sắp theo                Sắp xếp  

#169636 Đề thi HSG tinh Đông Tháp

Đã gửi bởi Prudential112410 on 18-10-2007 - 16:16 trong Thi HSG cấp Tỉnh, Thành phố. Olympic 30-4. Đề thi và kiểm tra đội tuyển các cấp.

Như thế là thiếu rồi!
Chú ý là$ a,b,c$ là số thực chứ không phải là số dương



#169635 Cho hỏi ...........

Đã gửi bởi Prudential112410 on 18-10-2007 - 16:13 trong Các dạng toán THPT khác

Hàm lõm $f''(x)>=0$



#166643 ChinaTST

Đã gửi bởi Prudential112410 on 13-09-2007 - 17:21 trong Tài nguyên Olympic toán

AI có các đề thi Trung Quốc không chia sẽ mình với!



#156403 Ai biết tổng quát nhe!

Đã gửi bởi Prudential112410 on 29-05-2007 - 09:41 trong Hàm số - Đạo hàm

Cho hàm số $y=2x^3-3(2m+1)x^2+6m(m+1)x+1$. Tìm $m$ để đồ thị hàm số có hai điểm cực trị đối xứng qua đường $y=x+2.$



#149002 Phương trình...

Đã gửi bởi Prudential112410 on 26-02-2007 - 16:26 trong Phương trình - hệ phương trình - bất phương trình

Thì các bác cứ giải ra kết quả luôn đi nói thì này thì ai nói chả được!
OK?



#148846 Phương trình...

Đã gửi bởi Prudential112410 on 25-02-2007 - 10:19 trong Phương trình - hệ phương trình - bất phương trình

Tìm $m$ để phương trình $m4^x-(2m+1)2^x+m+4=0$ có$ 2$ nghiệm trái dấu



#148845 Tính lim

Đã gửi bởi Prudential112410 on 25-02-2007 - 10:17 trong Dãy số - Giới hạn

$lim\dfrac{2\sqrt{4+x}-\sqrt[3]{64-x}}{x} (x-->0)$



#148843 Hàm số

Đã gửi bởi Prudential112410 on 25-02-2007 - 10:15 trong Hàm số - Đạo hàm

Cho hàm số © $y=\dfrac{x^2-x+1}{x-1}$
Xác định$ A(x;y) (x>1)$ thuộc © sao cho khoảng cách A đến giao điểm 2 tiệm cận là nhỏ nhất.



#148842 Khó xơi

Đã gửi bởi Prudential112410 on 25-02-2007 - 10:14 trong Tích phân - Nguyên hàm

Tính tích phân:
$I= \int\limits_{0}^{\dfrac{\pi}{4}}\dfrac{(1+tgx)^3}{(sinx+3cosx)^2}dx$



#148840 HTL

Đã gửi bởi Prudential112410 on 25-02-2007 - 10:12 trong Công thức lượng giác, hàm số lượng giác

Cho tam giác ABC, nhận dạng:
$8R^2sinAsinCsin(A+C)=\dfrac{b^2}{tg\dfrac{B}{2}}$



#142264 Cực trị hay và khó vớ i AM_GM

Đã gửi bởi Prudential112410 on 15-01-2007 - 15:32 trong Bất đẳng thức và cực trị

Holder là BĐT TQ của BĐT Bunhiacowski cho n bộ số.

Hay nói cách khác Holder bà Bunhiacopxki mở rộng.
OK



#139402 bDT CỦA TOANTHPT

Đã gửi bởi Prudential112410 on 23-12-2006 - 20:26 trong Bất đẳng thức và cực trị

Dùng Schur thế nào đây, đừng bắt anh phải dùng tuyệt chiêu nữa đấy!
hình như cũng hiệu quả đấy



#139194 bDT CỦA TOANTHPT

Đã gửi bởi Prudential112410 on 22-12-2006 - 15:00 trong Bất đẳng thức và cực trị

Cho x,y,z >0 CMR:

Bài này của "ông anh" mà, dùng BCS là ra thôi



#139000 bất đẳng thức

Đã gửi bởi Prudential112410 on 21-12-2006 - 15:55 trong Bất đẳng thức và cực trị

Chứng minh với mọi số thực dương a,b,c ta đều có:

Bài này dùng hệ số bất định hoặc là đặt ẩn phụ sao đó sử dụng thêm một BDT phụ nữa là ra.



#137744 Bất Đẳng Thức Qua Các Kỳ TS ĐH

Đã gửi bởi Prudential112410 on 14-12-2006 - 17:03 trong Bất đẳng thức và cực trị

Đây là hệ quả của Jensen mà ,chỉ cần c/m BDT sau là được
$\dfrac{1}{1+x^{2}}+\dfrac{1}{1+y^{2}}>=\dfrac{2}{sqrt{(1+x^2)(1+y^2)}}$
Ta CM:
$\sqrt{(1+x^2)(1+y^2)} \leq 1+xy$
Cài này thì khai triển là OK!



#137527 BT hình

Đã gửi bởi Prudential112410 on 13-12-2006 - 16:28 trong Hình học

Cho tam giác http://dientuvietnam...mimetex.cgi?ABC vuông tại http://dientuvietnam...mimetex.cgi?CI( http://dientuvietnam.../mimetex.cgi?AB) cuả góchttp://dientuvietnam.net/cgi-bin/mimetex.cgi?K thuộc http://dientuvietnam.../mimetex.cgi?AC). Tính http://dientuvietnam.net/cgi-bin/mimetex.cgi?\widehat{AKI}



#137290 Bất Đẳng Thức Qua Các Kỳ TS ĐH

Đã gửi bởi Prudential112410 on 12-12-2006 - 15:09 trong Bất đẳng thức và cực trị

[quote name='NPKhánh' date='December 09, 2006 11:38 am']Thoạt nhỉn cứ tưởng đây là bài toán tương tự như trên. Nhưng cũng tương tự thật
$\sum(sin^2a+1+cotg^2a)$
Bài này cũng rất hay!



#137289 Bất Đẳng Thức Qua Các Kỳ TS ĐH

Đã gửi bởi Prudential112410 on 12-12-2006 - 15:04 trong Bất đẳng thức và cực trị

Còn cách nào khác không .
Cách này quá mạnh, không phù hợp với các bạn thi ĐH!



#133765 Lửa là gì ?

Đã gửi bởi Prudential112410 on 27-11-2006 - 15:48 trong Những chủ đề Toán Ứng dụng khác

Để tạo thêm sự hào hứng mình xin hỏi một câu:
Vì sao nước lại có thể dập tắt được lưả?



#133522 Fanclub of MU

Đã gửi bởi Prudential112410 on 26-11-2006 - 15:14 trong Câu lạc bộ hâm mộ

Có tự tin quá không vậy



#132918 dễ lắm.

Đã gửi bởi Prudential112410 on 24-11-2006 - 11:44 trong Công thức lượng giác, hàm số lượng giác

Cho tam giác http://dientuvietnam...mimetex.cgi?ABC thỏa :Nhận dạng tam giác



#132917 Không dễ..1

Đã gửi bởi Prudential112410 on 24-11-2006 - 11:41 trong Công thức lượng giác, hàm số lượng giác

Cho tam giác ABC thỏa http://dientuvietnam...gi?3(cosB 2sinC)+4(sinB+2cosC)=15Nhận dạng tam giác http://dientuvietnam...mimetex.cgi?ABC



#131929 Bài hát bạn yêu thích nhất là gì?

Đã gửi bởi Prudential112410 on 21-11-2006 - 14:51 trong Quán nhạc

Các bác nge bài này xem có được không.Tui khoái nhất đấy :beta : D
Đừng Giận Anh Em Nhé.

ủa có thầy gì đâu nào bạn?



#131928 Một bài cực trị (có diễn đàn thảo luận rùi,mừng wá)

Đã gửi bởi Prudential112410 on 21-11-2006 - 14:48 trong Bất đẳng thức và cực trị

[quote name='Hannah Montana' date='November 21, 2006 11:15 am'] http://dientuvietnam...imetex.cgi?(a b)(1/a+1/b)>=4ta có : http://dientuvietnam.net/cgi-bin/mimetex.cgi?\dfrac{1}{2x+y+z}=\dfrac{1}{x+y+x+z}
Coi http://dientuvietnam...metex.cgi?a=x y
Cộng các vế BDT lại sử dụng gt ta có đpcm



#131597 Max

Đã gửi bởi Prudential112410 on 20-11-2006 - 08:26 trong Bất đẳng thức và cực trị

Bài này có lẽ sử dụng (a+b)(1/a+1/b)>=4 a,b>0$